LSAT and Law School Admissions Forum

Get expert LSAT preparation and law school admissions advice from PowerScore Test Preparation.

 UnicornChainsaw
  • Posts: 8
  • Joined: Mar 11, 2017
|
#35467
In lines 23-25 is says that certain plants would lose their "edge" given to them by photosynthetic efficiencies. Then the passage continues by saying that other plants will have increased performance. How do you get to answer "A" from there? It seems like the increased performance could be the result of some other factor.

Just because sugarcane and corn have decreased photosynthetic rates due to the CO2, why does that then infer that other plants have increased performance because of photosynthetic rates?? (I hope that makes sense ^_^ )

I am just having trouble following the logic of this answer.

Thank you for your help!
UC
 AthenaDalton
PowerScore Staff
  • PowerScore Staff
  • Posts: 296
  • Joined: May 02, 2017
|
#35519
Hi Unicorn,

The lines you highlighted mention that sugarcane and corn might lose their photosynthetic edge in a CO2-rich atmosphere "because of the increased performance of certain weeds." If the sugarcane and corn do lose some of their photosynthetic advantage to weeds, it makes sense that the weeds gain some of the photosynthetic advantage previously enjoyed by those crops.

Essentially, the corn and sugarcane won't be able to soak up as much sun as they usually do because the weeds that they compete with will be taking in a greater share of essential nutrients. The beneficial crops' loss is the weeds' gain.

From this we can conclude, as answer choice (A) does, that increasing CO2 levels will increase the photosynthetic rates of certain plants -- just not the ones that we want.

I hope this makes sense! Good luck studying.
 S2KMo
  • Posts: 12
  • Joined: Jun 10, 2018
|
#48018
Hello,

Could you please explain why D is wrong? I chose D because in the final paragraph the author writes "It is clear that the CO2 fertilization effect does not guarantee the lush world of agricultural abundance that once seemed likely..."

With this statement, it seemed as if it is directly highlighted answer choice D.

OR, is it incorrect because the author does not specifically address "abundances of certain species of plants"

Please help me rationalize this answer choice!!1
 Alex Bodaken
PowerScore Staff
  • PowerScore Staff
  • Posts: 136
  • Joined: Feb 21, 2018
|
#48047
S2KMo,

Thanks for the question! The reason that answer choice (D) is incorrect is that the author would agree (not disagree, as stated in answer choice [D]) that "enhanced plant growth could lead to abundances of certain species of plants" - she notes that "increased CO2 would differentially increase the growth rate of different species of plants" - meaning that some (aka certain) species of plants would grow more effectively, even as others grew less effectively. Because the author would agree with this statement, it cannot be the credited answer, which leaves us with answer choice (A)...which is close to a mirror of answer choice (D) and is therefore correct.

Hope that helps!
Alex
User avatar
 CristinaCP
  • Posts: 28
  • Joined: Sep 17, 2023
|
#104566
I'm confused as to why [D] is wrong. Aren't they saying that the effect of increased plant growth would be abundance for group 1 (the weeds) and yield reductions for group 2 (important crops) ? Don't they therefore disagree that increased plant growth could lead to abundances for group 2? [D] could be right if you take the "certain groups of plants" to be the important crops, and wrong if the "certain groups of plants" refers to the weeds. Is this ambiguity the reason why it's wrong?
 Robert Carroll
PowerScore Staff
  • PowerScore Staff
  • Posts: 1787
  • Joined: Dec 06, 2013
|
#104669
CristinaCP,

Your questions show why answer choice (D) is wrong. As you point out, certain plants will be more abundant, others less abundant. So there's nothing "incorrect" about the statement "enhanced plant growth could lead to abundances of certain species of plants." It could! The author agrees with that implication of the research in the first paragraph. It might not be the kind of abundance we want, as more weeds and less beneficial crop yields aren't good, but it's more abundant weeds.

Consider: "There are certain plants that would be more abundant if atmospheric CO2 levels increased." True! Weeds would be.

"There are certain plants that would not be more abundant if atmospheric CO2 levels increased." Also true - corn and sugarcane.

So it depends what "certain" refers to. We therefore can't call the statement "incorrect" without qualification.

Robert Carroll

Get the most out of your LSAT Prep Plus subscription.

Analyze and track your performance with our Testing and Analytics Package.